Biology praxis form 2 (8/13/22)

अब Quizwiz के साथ अपने होमवर्क और परीक्षाओं को एस करें!

Increases in which of the following have most contributed to the expansion of sand mining in the United States, especially in the upper Midwest, in recent years? A.Solar-energy operations B.Hydraulic-fracturing operations C.Wind-energy operations D.Beach-erosion mitigation

Correct Answer: B Option (B) is correct. High‑purity silica sand is particularly used to increase the porosity of the cracks blasted into shale bedrock by the high‑pressure pumping of water during hydraulic‑fracturing operations. The sand is suspended in the water, and once the water pumps are turned off, the water recedes but the sand remains and facilitates the flow of petroleum fluids from the well.

Induction of the lac operon in E. coli is most likely to occur under which of the following environmental conditions? Glucose Con. Lactose Con. A.Low Low B.Low High C.High Low D.High High

Correct Answer: B Option (B) is correct. The lac operon of E. coli encodes enzymes that enable the bacteria to take up the disaccharide lactose from the environment and hydrolyze the lactose to glucose and galactose. The operon is maximally active when the concentration of glucose is low in the environment and the concentration of lactose is high.

The table above lists some of the atomic characteristics of the element carbon. Which of the following statements about carbon is most relevant to understanding its role in the biochemistry of living organisms? A.Carbon has an equal number of protons‚ neutrons‚ and electrons. B.The number of electrons in the inner shell is unique to carbon. C.Carbon has four valence electrons, allowing it to bond with up to four other atoms in a variety of configurations. D.Carbon is a diatomic gas in its natural state.

Correct Answer: C Option (C) is correct. Carbon has four valence electrons, so rather than gaining or losing electrons, it typically forms four bonds to complete its outer electron shell. The bonds can be single, double, or triple bonds with other carbon atoms or single bonds with up to four other atoms. This provides for tremendous variability of sequence and structure among the molecules formed.

In mammals‚ fertilization normally occurs in which of the following locations? A.Placenta B.Uterus C.Oviduct D.Ovary

Correct Answer: C Option (C) is correct. In mammals, fertilization of an ovum by a sperm occurs in an oviduct. Fertilization is followed by the cleavage divisions of the developing embryo as the embryo passes along the oviduct toward the uterus. In humans, the oviducts are known as the fallopian tubes.

Which TWO of the following are typical characteristics of monocots but not of eudicots? A.Leaves with parallel venation B.Flower parts in multiples of four or five C.Vascular bundles in the stem arranged in a ring D.Adventitious roots

Correct Answer: A, D Options (A) and (D) are correct. Monocots have leaves with parallel veins, while the veins of eudicots typically occur in a netlike pattern. Monocots also have an adventitious root system of similarly sized roots that spread out from the stem, while eudicots typically have a main taproot from which other roots grow.

Convergent evolution is best represented by which of the following? A.The homology of forelimbs in whales and birds B.The streamlined body plan of dolphins and swordfish C.The development of placental mammals in a uterus and the development of marsupials in a pouch D.The similar embryonic development of reptiles and birds

orrect Answer: B Option (B) is correct. Convergent evolution refers to the independent evolution of similar features in organisms belonging to different lineages due to similar selection pressure. Dolphins, which are mammals, and swordfish independently evolved streamlined bodies that are adaptive for oceanic habitats.

The molecules of electron transport chains are found in which of the labeled locations of the organelle represented above? A.A B.B C.C D.D

Correct Answer: C Option (C) is correct. The molecules of the electron transport chain are embedded in the inner membrane, or cristae, of the mitochondria. The many folds of the cristae provide a great deal of surface area for both electron transport chain molecules and ATP synthase complexes.

Which of the following marine animals exhibits strictly bilateral symmetry?

Correct Answer: C Option (C) is correct. The only animal illustrated that exhibits strictly bilateral symmetry is the lobster. An animal with bilateral symmetry has distinct left and right sides.

Which of the following pathways is common to both fermentation and aerobic cellular respiration? A.Glycolysis B.Electron transport chain C.Photophosphorylation D.Citric acid cycle

Correct Answer: A Option (A) is correct. Glycolysis is a metabolic pathway that produces pyruvate from glucose. It occurs in the cytoplasm and is the first step in both fermentation and aerobic cellular respiration.

Contains vascular tissue Produces flagellated sperm Produces spores but not seeds Has separate gametophyte and sporophyte generations with a dominant sporophyte generation The characteristics of plant X are listed above. Plant X most likely belongs to which of the following taxa? A.Gymnosperms B.Bryophytes C.Pterophytes D.Angiosperms

Correct Answer: C Option (C) is correct. The listed characteristics are those of pterophytes. The phylum Pterophyta includes ferns, horsetails, and whisk ferns.

Which of the following is an example of endothermic heat production? A.A honeybee vibrating its flight muscles without moving its wings before taking flight B.A turtle diving off a floating log into the cool water of a lake on a hot day C.A vulture spreading its wings in the direction of the morning sun D.A desert lizard hunting for food early or late in the day but not at midday

Correct Answer: A Option (A) is correct. A honeybee that vibrates its flight muscles without moving its wings can successfully raise its body temperature several degrees above the ambient temperature before taking flight. This production of heat by muscle vibration is an endothermic behavior.

When initially placing a slide on a light microscope's stage, which of the following is the correct position of the microscope's stage and the correct magnification? Stage Position Magnification A.Low Low B.Low High C.High Low D.High High

Correct Answer: A Option (A) is correct. A slide should be placed on a microscope's stage when the stage is in a lowered position and with the lowest magnification objective lens in position. This arrangement helps to protect both the slide and the lens and is the most efficient way for the user to focus on the material on the slide.

Which of the following substances should be added to an acid spilled on a lab bench in order to safely neutralize the acid before cleaning up the spill? A.Sodium bicarbonate B.Potassium hydroxide C.Sodium chloride D.Ammonium sulfate

Correct Answer: A Option (A) is correct. A spill of most acids used in school laboratories can be neutralized by shaking sodium bicarbonate (baking soda) onto the spill. After any bubbling or fizzing has stopped, the pH p Hof the solution can be checked with pH p Hpaper. When neutralized, the spill should be carefully wiped up and the surface cleaned thoroughly.

Based on the phylogenetic tree shown, which of the following statements is most likely correct? A.Multicellularity evolved independently more than once. B.Fungi are more closely related to plants than to animals. C.There are more species of land plants than of red algae. D.Red algae and green algae can interbreed.

Correct Answer: A Option (A) is correct. All five groups shown at the tips of the phylogenetic tree include multicellular organisms. Because the common ancestor of the groups of organisms was unicellular, multicellularity must have evolved independently at least twice, once for the lineage that includes fungi and animals and once for the lineage that includes plants and algae. Multicellularity may have evolved up to five times, once per group, but this is less likely than that it evolved twice.

As a result of bacterial conjugation, the recipient bacteria are most likely to A.express a new phenotype B.produce bacteriophages C.ingest the donor bacteria D.become diploid for all chromosomal genes

Correct Answer: A Option (A) is correct. Bacterial conjugation occurs when a plasmid or a portion of the chromosome in one bacterium is transferred by the bacterium to a second bacterium, typically of the same species. The genes that are transferred during conjugation may cause the recipient bacterium to express a new phenotype.

Which of the following statements is most consistent with the survivorship curve shown for a population of soft‑shell clams (Mya arenaria) ? A.The M. arenaria population produced large numbers of fertilized eggs, but most did not result in organisms that survived past their first year. B.The M. arenaria population produced large numbers of fertilized eggs, and 50% of the resulting organisms survived to age 1 year. C.The M. arenaria population was initially above carrying capacity and decreased to carrying capacity by year 3. D.The M. arenaria population used up all resources by year 13 and went extinct in this area.

Correct Answer: A Option (A) is correct. Based on the curve, of the 10^4 organisms that arise from fertilized eggs for this particular population of M. arenaria, only about 10^2 clams actually survive to their first year. Mortality is about 99%. To correctly interpret the data, it is important to realize that the y‑axis represents a log scale.

Which of the following is most responsible for the movement of water up the stems of tall plants? A.Hydrogen bonding between water molecules B.The relatively high specific heat of water C.Organic solutes moving along concentration gradients D.Water being forced out of the plant by root pressure

Correct Answer: A Option (A) is correct. Because oxygen is more electronegative than hydrogen, the oxygen region of a water molecule has a partial negative charge, and the hydrogen regions have partial positive charges. The oppositely charged regions of adjacent water molecules are attracted to each other and form hydrogen bonds, so water molecules are held together in a relatively structured arrangement and are said to exhibit cohesion. Thus, as water molecules evaporate from the leaves of a plant, adjacent water molecules are essentially pulled up and along the stem of the plant toward the leaves at the top.

If cleavage of an embryo occurs in a radial fashion, the embryo must be of an organism that is A.a deuterostome B.a protostome C.an amniote D.a vertebrate

Correct Answer: A Option (A) is correct. Deuterostome development is characterized by radial cleavage of an embryo, where the cleavage planes are parallel and perpendicular to the axis of the embryo. This contrasts with a spiral cleavage pattern in protostomes. Deuterostomes include organisms in the phyla Chordata, Echinodermata, Hemichordata, and Xenacoelomorpha.

In a species of fish, some are dark gray (GG)‚ Capital G capital Gsome have light and dark gray stripes (Gg)‚ Capital G lowercase gand some are light gray (gg). lowercase g lowercase gA wildlife ecologist studying a particular population of the fish recorded the data above. Based on the data, which of the following is most likely true about the population? A.The population is not in Hardy-Weinberg equilibrium because the genotype frequencies of GG and gg are greater than the expected equilibrium frequencies. B.The population is not in Hardy-Weinberg equilibrium because 2pq 2 p qis not equal to p + q. C.The population is in Hardy-Weinberg equilibrium because in the heterozygotes the number of G alleles is equal to the number of g alleles. D.The population is in Hardy-Weinberg equilibrium because the number of G alleles in the population is equal to the number of g alleles.

Correct Answer: A Option (A) is correct. For a gene with only two alleles, p is the frequency of one allele in the gene pool of a population, and q is the frequency of the other allele; the sum of p + q must equal 1. The frequency of predicted genotypes for a non‑evolving population is given by the Hardy‑Weinberg equilibrium equation: p2 + 2pq + q2 = 1, is the frequency of organisms homozygous for one allele (e.g., GG fish), q2 is the frequency of organisms homozygous for the other allele (e.g., gg fish), and 2pq is the frequency of heterozygous organisms (e.g., Gg fish). In this population, p = 0.5 and q = 0.5. Based on the Hardy‑Weinberg equation, the population should include 250 dark gray fish (GG), 500 striped fish (Gg), and 250 light gray fish (gg). The population is not in Hardy‑Weinberg equilibrium because the predicted phenotype frequencies with respect to coloration are significantly different from the actual frequencies present in the population.

Gel electrophoresis of DNA D N Ais best used for which of the following applications? A.Separating DNA D N Afragments by size B.Cleaving DNA D N Amolecules at specific sequences C.Generating many copies of a particular DNA D N Asequence D.Hybridizing mRNA m R N Ato complementary genomic DNA D N Asequences

Correct Answer: A Option (A) is correct. Gel electrophoresis is used to separate proteins or nucleic acids by molecular weight. Electrophoresis of DNA D N Afragments is typically performed using agarose gels. In general, smaller DNA D N Afragments migrate further in the gel toward the positive electrode than do large fragments, which become trapped and slowed in the matrix of the gel.

In which of the following populations is genetic drift likely to have the greatest effect on the gene pool? A.A small population of tortoises on an island in the Pacific Ocean B.A small population of songbirds in a patch of deciduous forest several miles from another patch of forest with the same species of bird C.A large population of bats in the caverns of New Mexico D.A large population of rabbits in the fields of Virginia

Correct Answer: A Option (A) is correct. Genetic drift refers to any change in allele frequencies that is due to chance. It typically has the greatest effect on small, isolated populations where the random increase or decrease in the presence of a particular allele makes a relatively greater change in the frequency of that allele than it would in a large population. Any change in the frequency of a particular allele in a small population of tortoises isolated on an island will have a greater impact on the gene pool than will an allele frequency change in a small population with access to other members of the species or a large population of organisms where gene flow can occur.

Human skin color, height, and weight depend on the interaction of the products of multiple genes. These and other polygenic traits typically exhibit A.continuous variation B.complete dominance C.polyploidy D.pleiotropy

Correct Answer: A Option (A) is correct. Human skin color, height, and weight are influenced by many genes as well as environmental factors. When the frequency in a population of different values for each one of these traits is plotted on a histogram, a bell‑shaped curve of continuous variation usually results. The values of a few individuals are at either end of the curve; the values of most individuals are intermediate and fall in the middle of the curve.

The World Health Organization has declared that which of the following diseases has been completely eradicated from the world's population? A.Smallpox B.Yellow fever C.Polio D.Measles

Correct Answer: A Option (A) is correct. In 1979 the World Health Organization declared that smallpox was completely eradicated from the world's population. Vaccination, surveillance, and adoption of preventative measures to prevent viral spread from areas where smallpox was still endemic, primarily in Asia and Africa, led to the successful eradication of this viral infection.

In humans, deoxygenated blood enters the heart through the A.venae cavae B.aorta C.jugular veins D.pulmonary arteries

Correct Answer: A Option (A) is correct. In humans, deoxygenated blood enters the heart through the inferior vena cava that carries blood from the lower part of the body and the superior vena cava that carries blood from the upper part of the body including the head.

Studies of a particular lake revealed that for 3,000 to 4,000 years after the last glaciation material was deposited into the lake at a high rate. Then followed a long period during which the rate of deposition slowed to about one‑tenth of the previous rate. In the late 1800s eighteen hundredsthe deposition rate increased to about 100 times the intermediate rate. Which of the following most likely explains the large increase in deposition into the lake in the late 1800s?eighteen hundreds A.Human disturbances in the land surrounding the lake resulted in an increase of material flowing into the lake. B.Overfishing in the lake caused more material to drift to the bottom rather than being consumed by fish. C.Algal blooms resulting from climate change caused fish to die and drift to the bottom of the lake. D.Lack of rain in the area caused the lake to recede and the mineral content to become more concentrated.

Correct Answer: A Option (A) is correct. Increased land use and industrialization (e.g., of factories and farming practices) starting in the late 1800s eighteen hundredsmost likely contributed to the deposition of chemicals and other materials into the lake.

Based on the intermediate disturbance hypothesis, which of the following graphs best illustrates the relationship between the intensity or frequency of ecological disturbances and species diversity?

Correct Answer: A Option (A) is correct. It is hypothesized that an intermediate intensity or frequency of ecological disturbance maximizes species diversity. When disturbances are too rare, the species diversity in a region will be fairly stable, and if disturbances are too common or too severe, the region may contain few organisms of any species. At an intermediate intensity or frequency of disturbance, a region may contain organisms from both early and late successional stages.

Which of the following best describes the difference between natural selection and artificial selection? A.Environmental conditions drive natural selection, whereas humans determine artificial selection. B.Only some species are subject to natural selection, whereas all species are subject to artificial selection. C.Natural selection involves nature choosing a trait that is desirable, whereas artificial selection involves humans forcing the expression of one trait over another. D.Natural selection in a species occurs more rapidly than does artificial selection in the same species.

Correct Answer: A Option (A) is correct. Natural selection is driven by environmental pressures on organisms. Such pressures include the abundance of natural resources such as food, water, and shelter as well as the prevalence of predators. Artificial selection is driven by humans who breed particular organisms with desirable traits and select for progeny that exhibit the traits. Such breeding and selection are typically continued over many generations.

A diploid cell that produces haploid products by going through two consecutive rounds of nuclear division is best described as which of the following? A.A germ cell B.A cancer cell C.A stem cell D.A somatic cell

Correct Answer: A Option (A) is correct. Of the options, the only cells that are diploid cells that undergo two consecutive rounds of nuclear division to produce haploid gametes (the process of meiosis) are germ cells. In some usages, the gametes produced are also called germ cells.

Which of the following is generally true of ovum production in women? A.Oogenesis results in one large ovum and three small polar bodies. B.Ova are produced by mitosis of stem cells when a woman reaches reproductive age. C.Four functional ova are produced by the division of each primary oocyte, but only one ovum can be fertilized. D.Ovum production requires two cell divisions: one in which sister chromatids separate, followed by one in which the chromosome number is halved.

Correct Answer: A Option (A) is correct. Oogenesis results in one large ovum with most of the cytoplasm and three small polar bodies. During each of the two meiotic divisions, the metaphase plate forms to one side of the cell, causing acentric cellular division. This concentrates the organelles and nutrients required for metabolism by the ovum and for the early stages of embryonic development if the ovum is fertilized. The polar bodies typically disintegrate.

Nuclear envelope breakdown and chromosome attachment to spindle fibers is completed during which of the following stages of mitosis? A.Prometaphase B.Prophase C.Anaphase D.Metaphase

Correct Answer: A Option (A) is correct. Prometaphase is the phase of the cell cycle between prophase, when the chromosomes condense, and metaphase, when the chromosomes are aligned along the equator of the cell. During prometaphase, the condensed chromosomes use the motor proteins associated with their kinetochores to attach to and stabilize the spindle fibers, and the nuclear envelope breaks down.

A sample of peritoneal fluid from a laboratory mouse is placed between a slide and a 20 mm × 20 mm cover slip to count the number of viable white blood cells. The thickness of the peritoneal fluid sample between the slide and the cover slip is 0.1 mm. zero point one millimeterWhat is the volume of the sample? A.400 mm3 B. 40 mm3 C. 4 mm3 D. 0.40 mm3

Correct Answer: B Option (B) is correct. If the dimensions of the cover slip are 20 mm × 20 mm, the surface area of the sample is 400 mm2. If the thickness of the sample under the cover slip is 0.1 mm, then the total volume of the sample is 400 mm2 × 0.1 mm = 40 mm3.

Before the start of any laboratory exercise, which of the following safety precautions is most appropriate for a teacher to tell the students? A.Secure or remove articles of clothing that dangle and can get caught on lab equipment. B.Be aware of the locations of all electrical outlets that can cause a shock if used carelessly. C.Know the weight limits of the laboratory balances. D.Know the location of trash cans in case an object catches fire and must be thrown into a can to contain the fire.

Correct Answer: A Option (A) is correct. Students should be told to remove any articles of clothing that can get caught on lab equipment such as balances, microscopes, or centrifuges or that can dangle into an open flame or a caustic solution.

As the levels of carbon dioxide rise in Earth's atmosphere, which of the following is the most likely consequence? A.Trapping by greenhouse gases of the heat energy emitted by Earth's surface will increase, contributing to global warming. B.The ozone layer will be depleted, increasing exposure to mutation‑causing ultraviolet radiation. C.More solar radiation will enter the atmosphere, contributing to global warming. D.Cloud cover will increase, causing the average amount of yearly precipitation to rise.

Correct Answer: A Option (A) is correct. Sunlight passes through the atmosphere to warm the surface of Earth. Much of the heat striking Earth is radiated back toward space, but CO2 C O 2and other greenhouse gases trap much of this heat and then radiate it back to Earth, raising the temperature of Earth's surface. An increase in the concentration of atmospheric CO2 C O 2means that the amount of heat radiating back to Earth will increase and will thus contribute to global warming.

Which of the following events most directly addressed the protection of habitats and of threatened wildlife throughout the United States? A.Passing of the Endangered Species Act of 1973 B.Creation of the United States Fish and Wildlife Service C.Creation of the National Oceanic and Atmospheric Administration D.Establishment of the National Park Service

Correct Answer: A Option (A) is correct. The Endangered Species Act was passed in 1973 to protect species that are endangered or threatened throughout all or much of their range and to conserve ecosystems of which they are a part.

Hormone H binds to both liver and lung cells. It causes liver cells to release glucose but does not cause any response by lung cells. Of the following, which is the most likely explanation? A.Hormone H binding activates a glucose transporter gene‑specific transcription factor in liver cells but not in lung cells. B.Hormone H is encoded by the genome of liver cells but not by the genome of lung cells. C.Hormone H encodes a glucose channel protein that is translated in liver cells but not in lung cells. D.Hormone H binding blocks G‑protein activation in lung cells but not in liver cells.

Correct Answer: A Option (A) is correct. The binding of a hormone to its receptor on the plasma membrane of a cell typically signals the cell to make a specific response. If the binding of hormone H to both liver and lung cells causes only the liver cells to release glucose, then it is likely that hormone H causes the expression or activation of transcription factors in liver cells, but not lung cells, that bind to the regulatory sequences of a glucose transporter gene and trigger expression of a plasma membrane glucose‑transporter protein.

Which of the following is a reason why the use of genetically modified organisms (GMOs) G M O'sas food crops is controversial? A.The GMOs G M O'smay contain new substances that may not be safe for all consumers. B.The GMOs G M O'scan facilitate the development of new forms of plant disease. C.The genetic changes in the GMOs G M O'scan be passed on to many unrelated species. D.Extra genes in GMOs G M O'splace them at a competitive disadvantage with unmodified organisms.

Correct Answer: A Option (A) is correct. The general public has been concerned that genetically modified food crops might have been altered in such a way as to contain new substances that are potentially harmful to some consumers. So far, the data from research performed by reliable, independent major health groups worldwide support the safety of the crops for all consumers.

Based on the data graphed above‚ which of the following statements is correct? A.The graph represents data that may allow scientists to propose a theory. B.The graph represents data that were collected to prove a hypothesis. C.The graph represents data that were collected to propose a law. D.The graph represents data that may allow scientists to prove a theory.

Correct Answer: A Option (A) is correct. The graph represents data that may allow scientists to propose a theory. A theory is a well‑developed explanation of a scientific phenomenon or related phenomena that has been repeatedly confirmed through observation or experimentation. The data in the graph might be used to support the theory that recent increases in the atmospheric concentration of CO2 C O 2are contributing to the climate change that has become so apparent today.

The largest reservoir of an important nutrient is sedimentary rock. Mycorrhizae associated with the roots of a plant assist the plant in the uptake of the nutrient from the soil. The leaves of the plant are eaten by primary consumers‚ and the nutrient is incorporated into the organisms' DNA. D N ADecomposers return the nutrient to the soil. The nutrient is most likely part of which of the following cycles? A.Phosphorus cycle B.Carbon cycle C.Water cycle D.Nitrogen cycle

Correct Answer: A Option (A) is correct. The nutrient is part of the phosphorus cycle. The largest reservoir of phosphorus is in sedimentary rock; there is no appreciable atmospheric component to the phosphorus cycle. The weathering of rocks adds phosphate (PO43−) P O 4 with a negative 3 chargeto the soil. Mycorrhizae, symbiotic relationships between fungi and the roots of plants, help plants take up the phosphate from the soil, and the phosphate is incorporated into molecules of the plants that are in turn eaten by consumers, where it is again incorporated into biological molecules such as DNA. D N ADecomposers degrade dead producers, consumers, and feces and so return phosphorus to the soil, where much of it is typically incorporated into relatively insoluble compounds.

Guard cells, which control the opening and closing of stomata on the surface of leaves, are most important for which of the following reasons? A.They regulate water loss. B.They capture nitrogen from the atmosphere. C.They expel phloem sap that has reached the leaves. D.They absorb water from rain that can be stored in the roots.

Correct Answer: A Option (A) is correct. The primary function of the pair of guard cells that flank each stoma is to regulate the amount of water lost from a plant. Guard cells increase in turgor pressure and cause an increase in the size of the stomatal opening when the guard cells accumulate K+ potassium ionsfrom neighboring cells, which in turn causes osmotic flow of water into the guard cells. When the guard cells lose K+ potassium ionsand thus water, they become flaccid, leading to closure of the stomatal opening and preventing water loss through the stomata.

A scientist examining fossils of small rodents at a particular location found fossils of one species in a deep stratum and found fossils of a similar but distinct more recent species in the stratum immediately above. Fossils of transitional organisms were not found. Of the following, which is the best explanation of the observed distribution of fossils in the strata? A.There was a brief and rapid time of evolutionary change in the older species, so few fossils of transitional organisms exist. B.The more recent species outcompeted the older species, causing the older species to become extinct. C.The newer species invaded the area and interbred with the older species. D.The two species underwent sympatric speciation.

Correct Answer: A Option (A) is correct. The theory of punctuated equilibrium proposes that species are generally evolutionarily stable for a long period of time, and thus there is the likelihood of fossil formation. On rare occasions, a population of a species undergoes rapid evolutionary change, usually resulting in the formation of a new species. The new species will then be evolutionarily stable for a long period of time and likely to contribute to the fossil record. Because of the rapid evolution of earlier species to new species, few fossils of transitional species are found.

The growth hormone (GH) receptor is a cell surface transmembrane protein with a cytosolic domain that becomes phosphorylated in response to growth hormone binding to the extracellular domain. Which of the following is correct about the location of the GH receptor protein during its synthesis? A.The GH receptor is initially embedded in the endoplasmic reticulum membrane with its phosphorylation sites on the cytosolic surface and the GH‑binding sites in the ER lumen. B.The GH receptor is initially embedded in the endoplasmic reticulum membrane with its GH‑binding sites on the cytosolic surface and the phosphorylation sites in the ER lumen. C.The entire GH receptor protein is initially located in the lumen of the endoplasmic reticulum. D.The entire GH receptor is initially located in the cytosol of the cell, where it remains until it is inserted in the plasma membrane.

Correct Answer: A Option (A) is correct. Translation of proteins such as the GH G Hreceptor that will be embedded in the plasma membrane is carried out by ribosomes on the cytosolic surface of the rough endoplasmic reticulum (ER). E RDuring translation of the protein, it starts (from its amino‑terminus end) to enter the ER E Rthrough pores in the ER E Rmembrane. The amino terminus of the protein is typically the domain to which a ligand such as GH will bind. Insertion of the protein into the ER E Ris halted when amino acids of the transmembrane portion of the protein form bonds with components of the ER E Rmembrane; the cytosolic domain of the protein that contains the phosphorylation sites remains in the cytosol of the cell. Vesicles that contain the receptor protein pinch off the ER E Rand fuse with the membrane of the Golgi apparatus such that the extracellular ligand‑binding domain of the receptor is located in the cisternae of the Golgi apparatus, while the cytosolic domain remains in the cytosol. When vesicles containing the receptor pinch off the Golgi apparatus and fuse with the plasma membrane, the receptor becomes incorporated into the membrane with the ligand‑binding domain on the extracellular surface of the cell and the cytosolic phosphorylation sites in the cytosol.

Which of the following best explains why the body makes a more rapid and stronger response to an infecting pathogen after vaccination against the pathogen? A.Memory lymphocytes made in response to the vaccination are activated. B.Preformed antibodies made in response to the vaccination are stored in the spleen. C.The number of lymph nodes in the body increases in response to the vaccination. D.The material used for the vaccination is stored in the lymph nodes and spleen until needed during an infection.

Correct Answer: A Option (A) is correct. Vaccination against a pathogen activates naïve lymphocytes with receptors specific to that pathogen. Activation of the naïve lymphocytes is slow, but once activated, they produce clones of cells with identical pathogen specificity. The clones include lymphocytes referred to as memory cells. The memory cells remain in the individual and, in contrast to naïve lymphocytes, are rapidly activated in response to an infection by the same pathogen.

Which of the following shows the correct route for the movement of water into a root of a plant? A.Root epidermis → → cortex → → endodermis → → pericycle → → xylem B.Root epidermis → → endodermis → → cortex → → pericycle → → xylem C.Root epidermis → → pericycle → → cortex → → endodermis → → xylem D.Root epidermis → → endodermis → → pericycle → → cortex → → xylem

Correct Answer: A Option (A) is correct. Water and minerals absorbed from the environment enter a root of a plant through the epidermis, then pass through the cells that make up the cortex, then through the single‑cell‑thick endodermis, and then through the thin pericycle cell layer to finally reach the xylem vessels.

Which of the following best explains why an article in a free online encyclopedia is generally considered to be an unreliable scientific source? A.The article is not peer reviewed. B.Many people contribute to the writing of the article. C.Some people may disagree about the content of the article. D.The article can be modified over time.

Correct Answer: A Option (A) is correct. When a scientific paper is submitted to a reputable journal for publication, it is critically reviewed by other scientists who are familiar with the field of research before the paper is accepted for publication. Based on the review, the paper may also be rejected. Free online encyclopedias do not typically require peer reviews, so the articles may be factually incorrect.

Frequent low-intensity wildfires are most likely to have which TWO of the following ecological benefits? A.Recycling of nutrients B.Removal of dead underbrush C.Formation of new topsoil from exposed rocks D.Establishment of shade-tolerant plant species

Correct Answer: A, B Options (A) and (B) are correct. Dry, dead underbrush is particularly susceptible to burning by frequent low‑intensity wildfires. The burning of the underbrush opens space for seeds to germinate and recycles nutrients back into the soil that are beneficial to all other plants.

By which TWO of the following mechanisms is most of the nitrogen required by terrestrial plants made available to the plants? A.Fixation of atmospheric nitrogen by soil bacteria including those associated with the roots of plants B.Decomposition of organic material in the soil by bacteria and fungi C.Direct absorption of atmospheric nitrogen through the stomata of the plants D.Decomposition of nitrogen‑containing compounds by plant‑secreted enzymes around the roots of the plants

Correct Answer: A, B Options (A) and (B) are correct. Plants require nitrogen for the synthesis of nucleic acids and proteins, but they cannot fix atmospheric nitrogen (N2). N 2They are dependent on bacteria and fungi to make nitrogen available in a form they can use. Soil bacteria, which are often associated with the roots of plants, fix atmospheric nitrogen into ammonia (NH3), N H 3and bacteria and fungi also decompose organic material in the soil and release NH3. N H 3Most of the NH3 N H 3is modified through a couple of chemical reactions into nitrate (NO3−) N O 3 with a negative chargethat can be absorbed by the roots of plants.

To study the inheritance of wing size in fruit flies, true-breeding female flies with large wings were crossed with male flies with miniature wings (Cross 1). The F1 F 1progeny were then crossed. The reciprocal parental cross (Cross 2) was also performed, and the F1 F 1progeny were crossed. The crosses and the results are summarized in the table above. Based on the data, which TWO of the following statements are most likely correct? A.The gene for wing size is sex-linked. B.Homozygous expression of the allele for miniature wings is frequently lethal during embryogenesis. C.The allele for miniature wings is recessive to the allele for large wings. D.The two alleles for wing size are codominantly expressed in female flies.

Correct Answer: A, C Options (A) and (C) are correct. The discrepancy in wing size ratios between the female and male F2 F 2progeny of the F1 ×× F1 F 1 by F 1mating from Cross 1 and between the female and male F1 F 1progeny of Cross 2 suggests that the gene for wing size is sex‑linked. The data are explained if females have two copies of the wing size gene and males have only one copy and if the allele for miniature wings is recessive to the allele for large wings. The fact that the F1 F 1progeny from Cross 1 all have large wings and that none of the crosses produce progeny that have only miniature wings supports the statement that the allele for miniature wings is recessive to the allele for large wings.

Which TWO of the following pose the greatest obstacle to the random passive diffusion of ions across the plasma membrane of a cell? A.The hydrophobic interior of the plasma membrane's lipid bilayer B.The high concentration of solutes in the cell's cytosol C.The repulsion of ions by the polar heads of phospholipids D.The large size of ions that are hydrated by water molecules

Correct Answer: A, D Options (A) and (D) are correct. Charged ions are hydrophilic, but the phospholipid fatty acid tails in the interior of a membrane are uncharged and hydrophobic, so ions are unable to diffuse between the fatty acid tails and traverse the plasma membrane of a cell. Because the ions are hydrophilic, each ion is also surrounded by a shell of water molecules (hydrated), making the ion quite large. The large size of the hydrated ions also impedes their diffusion across the membrane. Ions typically require a transmembrane protein channel with a hydrophilic interior to facilitate their diffusion across the membrane.

Population : Community Which of the following is most analogous to the organizational relationship above? A.Cell : Organelle B.Tissue : Organ C.Allele : Gene D.Polypeptide : Amino acid

Correct Answer: B Option (B) is correct. A community is composed of many populations of interacting organisms. Similarly, an organ is composed of several types of tissues that work together and enable the organ to perform its function.

Which of the following is true of peptide bond formation in a lengthening polypeptide? A.Many ribosomes can simultaneously bind to a polypeptide and form peptide bonds. B.As a peptide bond forms, the polypeptide is transferred from the peptidyl (P) site to the aminoacyl (A) site of the ribosome. C.The binding of two tRNAs t R N A'sto each other brings the polypeptide and a new amino acid into close proximity. D.A peptide bond is synthesized by protein components of the ribosome.

Correct Answer: B Option (B) is correct. A growing polypeptide is initially held in the P site of a ribosome. After a tRNA t R N Acarrying the next amino acid required for the polypeptide binds to the corresponding codon in the A site of the ribosome, a peptide bond forms between the polypeptide and the amino acid. Bond formation releases the polypeptide from the tRNA t R N Ain the P site and transfers it to the A site.

The H+ concentration of a solution is 10-4 M. ten to the negative four MolarBy how many times must the H+ concentration decrease for the solution to reach a pH of 7.0 ? A.4 B.10^3 C.4 × 10^3 D.10^4

Correct Answer: B Option (B) is correct. A solution with a pH of 7.0 has an H+ concentration of 10-7. The difference in magnitude between a concentration of 10-4and a concentration of 10-7 is 10^3.

Biopsies of malignant tumors often include a karyotype, and the results sometimes reveal a chromosomal translocation. Which of the following best explains why translocations can cause cancer? A.Translocations can cause the loss of large parts of chromosomes from a cell, which causes cell death. B.Translocations can result in the coding sequence of a gene being adjacent to the regulatory sequence of a different gene. C.Translocations can disrupt mitosis and cytokinesis, which results in polyploid daughter cells. D.Translocations can result in decreased genetic variation when they occur between homologs during crossing-over.

Correct Answer: B Option (B) is correct. A translocation occurs when the end of a chromosome breaks off and becomes joined to another chromosome or when the ends of two chromosomes break off and join to the reciprocal chromosomes. If the break separates the coding sequence of an inactive gene from its regulatory regions, the coding sequence may join to another chromosome adjacent to a regulatory sequence to which many transcription factors bind, so that the previously inactive gene is now actively transcribed. Such translocations often involve genes that control the rate of cell division, so the cell with a translocation may start to divide much more frequently, which is a characteristic of cancer cells.

Which of the following is most likely to increase transcription of a particular gene? A.More rapid pre‑mRNA m R N Aprocessing B.Acetylation of histone proteins associated with the gene C.Methylation of the gene and adjacent DNA D N Asequences D.Binding of a ribosome to the start codon

Correct Answer: B Option (B) is correct. Acetylation of the histone proteins associated with a particular gene facilitates transcription of the gene. DNA D N Ais negatively charged because of its phosphate groups and so readily bonds with positively charged histone proteins that contribute to the nucleosome structure of DNA D N Ain eukaryotic cells. When DNA D N Ais tightly bound by histone proteins, the transcription machinery of a cell cannot gain access to the DNA, D N Aand transcription is blocked. However, many transcription factors are associated with proteins that add negatively charged acetyl groups to the histones, which loosens the interactions between the DNA D N Aand histones and thus facilitates transcription of particular genes.

Which of the following are found in all viruses? A.An envelope and either DNA or RNA B.A capsid and either DNA or RNA C.A capsid, an envelope, and RNA D.A capsid and DNA

Correct Answer: B Option (B) is correct. All viruses have a genome composed of either DNA or RNA that is surrounded by a capsid that is typically composed of many copies of one or just a few proteins. Only some viruses have an additional envelope surrounding the capsid.

Scientists predict that a particular coastal village in Alaska will be underwater by the year 2025. Which of the following is most likely contributing to the submergence of the village? A.The village is sinking as a result of plate tectonics. B.Global climate change is thinning Arctic sea ice and raising sea levels. C.Natural variations in thermohaline circulation are changing tidal patterns. D.Carbon dioxide is decreasing the pH p Hof seawater and destroying barrier reefs.

Correct Answer: B Option (B) is correct. Arctic sea ice is thinning at an alarming rate, contributing to the rise of sea levels. Based on the rate at which sea levels are rising, coastal areas that are not much above sea level are predicted to be covered very soon by water

In angiosperms, fruits are formed from which of the following? A.Ovule B.Ovary C.Endosperm D.Embryo

Correct Answer: B Option (B) is correct. As seeds develop in the ovary of an angiosperm, the wall of the ovary thickens to protect the seeds. The fruit is the mature seed‑bearing ovary.

The figure above illustrates the mass of Atlantic cod caught in the Grand Banks area off the coast of Newfoundland from 1850 to 2010. Based on the data, which of the following most likely explains the 1975 collapse of the Atlantic cod fishing industry in this region? A.The bioaccumulation of anthropogenically introduced mercury to toxic levels B.The fishing of the population at unsustainable levels C.An increase in the number of icebergs carried by the Labrador Current past the Newfoundland coast D.A decrease in ocean acidity to levels intolerable by the population

Correct Answer: B Option (B) is correct. Based on the data shown, the 1975 collapse of the Atlantic cod fishing industry in the region most likely resulted from fishing of the population at unsustainable levels. In each of the ten to fifteen years before the collapse, the mass of cod captured in a single year was up to seven times the mass captured in a single year during the mid‑1800s.

Behavioral isolation is most likely to occur if two species differ in which of the following? A.Habitat ranges B.Courtship displays C.Chromosome number D.Chemical compatibilities of gametes

Correct Answer: B Option (B) is correct. Behavioral isolation provides a reproductive barrier to the hybridization of different species. Species-specific courtship displays, probably the most common form of behavioral isolation, enable mate recognition within a species but are not recognized by members of another species.

The table above shows observations made on four different cell types. Based on the data‚ of the cell types, which is most likely prokaryotic? A.W B.X C.Y D.Z

Correct Answer: B Option (B) is correct. Cell type X is most likely prokaryotic. Prokaryotes typically have cell walls and a single chromosome. Some prokaryotes, such as cyanobacteria, also have photosynthetic pigments.

Which of the following is a plant adaptation selected for by frequent natural wildfires in some ecosystems? A.Vascular bundles in the leaves B.Cones that open at high temperatures C.Shallow root systems D.Stomata on the lower surface of the leaves

Correct Answer: B Option (B) is correct. Certain gymnosperms typically release the seeds from their cones only after exposure to high temperatures such as those produced by a fire. Many of these trees are native to areas that experience frequent natural wildfires. The heat from the fires removes the resin and other material that seals the seeds under the scales of the cone, releasing the seeds that then germinate in the rich soil produced by the decomposition of trees and brush burnt by the fires.

Transmissible spongiform encephalopathies (TSE) T S Eare a family of neurodegenerative disorders caused by prions. Prions are infectious misfolded protein sequences that can enter cells and alter the folding of normal cellular prion proteins. A student claims that prions are living organisms because they are found in cells, can reproduce, and respond to the environment. Which of the following statements about prions best refutes the student's claim? A.Prions are structurally stable and are resistant to denaturation. B.Prions have no cellular machinery with which to carry out metabolism. C.Prions are unable to survive independently. D.Prions lack structures for movement.

Correct Answer: B Option (B) is correct. Characteristics of living organisms include the enclosure of their cells by a membrane, genomes composed of DNA, D N Aand the presence of cellular machinery with which to harness a source of energy and carry out metabolism. Prions are misfolded proteins whose interaction with normal prion proteins found in cells causes the proteins to fold into a similar misfolded shape. Prions are not cells and cannot carry out any metabolic processes.

The petals of a flower are known collectively as the A.calyx B.corolla C.stamen D.pistil

Correct Answer: B Option (B) is correct. Collectively, all the petals of a flower are called the corolla. The corolla generally surrounds the reproductive parts of a flower and may help attract pollinators.

If populations of two species are at carrying capacity and the species exhibit a commensal relationship, which of the following graphs shows the most likely change in the population sizes of the two species when they are isolated from each other at time equals 0 ?

Correct Answer: B Option (B) is correct. If populations of two species have a commensal relationship, then one species benefits from the presence of the second species, and the second species is unaffected by the presence of the first. If the two populations are suddenly isolated from each other, only the species that is dependent on the presence of the second is likely to be negatively affected and show a decline in population size, as is observed for species 1 in option (B). The population size of species 2 is unchanged.

In corn, the allele for purple kernel color is dominant over the allele for yellow kernel color. A purple kernel is planted, and the resulting plant is pollinated from a pure-breeding plant with yellow kernels. If the purple kernel is heterozygous, the phenotype ratio of the progeny is expected to be approximately A.1 purple : 0 yellow1 purple to 0 yellow B.1 purple : 1 yellow1 purple to 1 yellow C.3 purple : 1 yellow3 purple to 1 yellow D.9 purple : 7 yellow9 purple to 7 yellow

Correct Answer: B Option (B) is correct. If purple kernel color (Y) capital Yin corn is dominant over yellow kernel color (y), lowercase ythen kernels will be purple if they are either homozygous for purple color (YY) capital Y capital Yor heterozygous (Yy). capital Y lowercase yIf a heterozygous plant with purple kernels (Yy) capital Y lowercase yis pollinated from a plant with yellow kernels (yy), lowercase y lowercase yabout one‑half of the progeny plants will be heterozygous plants with purple kernels, and one‑half will be plants with yellow kernels.

Lactated Ringer's is a salt solution that is used by physicians and veterinarians to restore fluid levels in the body in instances of injury or illness. A 500 mL preparation of Lactated Ringer's includes 3 g of NaCl. If the formula weight of NaCl is 58.44, the molarity of NaCl in Lactated Ringer's is approximately A.0.05 M B.0.1 M C.0.2 M D.1.0 M

Correct Answer: B Option (B) is correct. If the formula weight of NaCl is 58.44, 1 L of 1 M NaCl contains approximately 60 g of NaCl, and 500 mL of 1 M NaCl contains approximately 30 g of NaCl . Thus if 3 g K.O.M of NaCl grams are included in 500 mL of Lactated Ringer's, the molarity of NaCl N A C Lin this salt solution must be approximately 0.1 M.

C6H12O6 + 6 O2 → 6 CO2 + 6 H2O + Energy Shown above is the summary equation for aerobic cellular respiration. Which of the following indicates the molecule that is oxidized and the molecule that is reduced as the process proceeds? Oxidized. Reduced A.C6H12O6. CO2 B.C6H12O6 O2 C.O2 CO2 D.O2 H2O

Correct Answer: B Option (B) is correct. In the summary formula shown for aerobic cellular respiration, C6H12O6 is oxidized and O2 is reduced. The electrons of H atoms removed from C6H12O6 during glycolysis and the citric acid cycle reduce O2, the final electron acceptor of each electron transport chain, to produce H2O.

Which of the following is most consistent with the concept of green building as certified by the United States Leadership in Energy and Environmental Design (LEED) program? A.Camouflaging structures with paints and plants so that the structures blend into the landscape B.Building and maintaining structures using resource‑efficient, minimally polluting practices C.Surrounding structures with landscaping that uses only plants native to the area D.Building structures of primarily natural materials and including roof gardens

Correct Answer: B Option (B) is correct. LEED‑certified buildings are resource efficient. They are designed to reduce the use of water and energy and reduce greenhouse gas emissions. The certification may be applied to almost any type of building, and the building may be new construction or a renovation.

Which of the following best indicates the modes of digestion typically used by most heterotrophic protists and animals? Heterotrophic Protists Animals A.Extracellular Intracellular B.Intracellular Extracellular C.Extracellular Extracellular D.Intracellular Intracellular

Correct Answer: B Option (B) is correct. Most heterotrophic protists use endocytosis to ingest food from the environment into their cells, where it is digested, usually in lysosomes. Animals typically have an extracellular compartment for digestion. Many animals with simple body plans have a gastrovascular cavity that has a single opening. More complex animals have an alimentary canal with an opening at each end. Once the food is digested down to macromolecule size in the extracellular compartment, the macromolecules are brought into the cells as building blocks for new molecules and an energy source for cellular respiration.

Some vascular plants produce defensive chemicals, such as nicotine and tannins, that deter herbivores. These chemicals are primarily stored in which of the following components of plant cells? A.Nucleolus B.Vacuole C.Mitochondrion D.Cytosol

Correct Answer: B Option (B) is correct. Most plant cells have a large central vacuole and may also have many smaller vacuoles. One of the many functions of plant vacuoles is to store ions and assorted metabolites including defensive compounds such as nicotine and tannins

Which of the following experimental methods is most appropriate for separating pigments extracted from plant leaves? A.Gel electrophoresis B.Chromatography C.Spectrophotometry D.Distillation

Correct Answer: B Option (B) is correct. Pigments extracted from plant leaves are typically separated from each other by chromatography. A drop of liquid from an extract of ground leaves is placed on a strip of chromatography paper close to the bottom, and the very bottom of the paper, below the extract drop, is placed in an organic solvent. As the solvent travels up the strip, the pigments in the extract separate based on their solubility in the solvent and their attraction to the hydrated cellulose in the chromatography paper.

Which of the following treatments will most likely cause cells to burst and release their contents? A.Placing the cells in a drying oven B.Placing the cells in distilled water C.Placing the cells in an isotonic salt solution D.Placing the cells in a hypertonic salt solution

Correct Answer: B Option (B) is correct. Placing cells in distilled water will cause the cells to burst (lyse) and release their contents. Cells contain many molecules and are thus hyperosmotic to distilled water that typically contains none. Because the plasma membrane of cells is freely permeable to water but not the molecules, there will be a net flow of water into the cells, causing them to swell and burst.

Of the following‚ which was a result of the Miller‑Urey experiment that suggested an early event in the emergence of life on Earth? A.Self‑replication of RNAR N A B.Spontaneous assembly of organic molecules from inorganic molecules C.Spontaneous assembly of polypeptides from free amino acids D.Spontaneous formation of phospholipid membranes around free‑floating organic molecules

Correct Answer: B Option (B) is correct. The Miller‑Urey experiment combined CH4, NH3, and H2 above a flask of boiling water as a source of water vapor and used electrical sparks as an energy source. After a period of time, the water became cloudy and red and contained carbon‑containing (organic) compounds such as hydrogen cyanide (HCN) and formaldehyde (H2CO) and some simple amino acids. Both HCN and H2CO can be used in the synthesis of more complex organic compounds.

Which of the following is the best way to assess the validity of an experiment? A.Statistical analysis of the results B.Reproducibility of the results C.Alignment of data to the hypothesis D.Reduction of experimental error

Correct Answer: B Option (B) is correct. The best way to assess the validity of an experiment is to reproduce the results. One reason for providing a detailed methods section in scientific papers is so that other scientists can follow the same procedure and test the reproducibility of the data.

The graph above shows the population size of chipmunks over time in a particular forest. An abiotic factor that could have resulted in the data at the bracketed portion of the graph is most likely which of the following? A.The introduction of wolves B.An outbreak of a forest fire C.The release of nonnative chipmunk species D.An infectious disease caused by a virulent pathogen

Correct Answer: B Option (B) is correct. The bracketed portion of the graph indicates a sudden rapid reduction in the size of the chipmunk population. An abiotic (nonliving) factor that could have been responsible for the rapid reduction in population size is the outbreak of a forest fire.

Sulfur oxides such as sulfur dioxide (SO2) S O 2and sulfur trioxide (SO3) S O 3are released by the burning of coal. Which of the following is the most direct significant environmental impact of the gases? A.Global warming B.Formation of acid precipitation C.Alteration of the jet stream D.Destruction of the ozone layer

Correct Answer: B Option (B) is correct. The burning of fossil fuels such as coal releases sulfur oxides into the atmosphere. These gases react with water in the atmosphere and form weak solutions of sulfuric acid. This increases the acidity of precipitation. Acid precipitation makes waters acidic, changes the composition of soil, and negatively affects large numbers of plants and animals.

The yucca plant cannot be pollinated and produce seeds without the presence of a small white moth called a yucca moth. The yucca moth in turn is entirely dependent on the yucca plant for its own reproduction. The interdependence of the plant and the moth exemplifies A.convergent evolution B.coevolution C.divergent evolution D.microevolution

Correct Answer: B Option (B) is correct. The dependence of the plant and the moth on each other for their reproduction suggests that the two organisms evolved together over a long period of time and each influenced the evolution of the other.

A male mouse with black fur and a short tail mates with a female mouse that has brown fur and a long tail. The offspring include pups that resemble the parents, as well as some that are brown with short tails and black with long tails. The appearance in the pups of these two phenotypes that differ from the parental phenotypes is best explained by which of the following? A.The principle of segregation B.The principle of independent assortment C.Replication during interphase D.Nondisjunction during meiosis

Correct Answer: B Option (B) is correct. The offspring show all four possible phenotypes with respect to fur color and tail length. This suggests that the alleles of the two different genes, fur color and tail length, are on different chromosomes and are transmitted independently, following the principle of independent assortment.

The human population on Earth has grown from an estimated one billion individuals in 1800 to over seven billion individuals. Which of the following best describes human population growth since 1800 ? A.The change in the number of individuals has been constant over time. B.The growth rate is exponential. C.The death rate exceeds the birth rate. D.The population is at carrying capacity.

Correct Answer: B Option (B) is correct. The rapid increase in the human population on Earth of approximately six billion individuals since 1800 suggests an exponential growth rate over this period of time.

Of the following biomes‚ which is best characterized by a consistent climate‚ nutrient‑poor soil‚ and high biodiversity? A.Grassland B.Tropical rain forest C.Boreal forest D.Temperate forest

Correct Answer: B Option (B) is correct. The tropical rain forest biome is characterized by a consistently warm, wet climate, a thin layer of nutrient‑poor soil, and very high biodiversity.

Which of the following best identifies the bonds between the components of a DNA molecule? Bonds Btwn Strands Bonds Btwn Backbone Components A.Covalent. Hydrogen B.Hydrogen Covalent C.Hydrogen Ionic D.Ionic Covalent

Correct Answer: B Option (B) is correct. The two strands of a DNA molecule are held together by hydrogen bonds between the nitrogenous bases. The alternating deoxyribose and phosphate groups of the backbone are linked by covalent phosphoester bonds (and sequential nucleotides are thus linked by phosphodiester bonds).

Which of the following properties of water is responsible for the solubility of many molecules in aqueous fluids such as blood and urine? A.The low density of solid water in comparison to liquid water B.The polarity of water molecules C.The high specific heat of water D.The approximately neutral pH of water

Correct Answer: B Option (B) is correct. Water molecules are polar because oxygen is more electronegative than hydrogen. The oxygen region of a water molecule has a partial negative charge, and the hydrogen regions have partial positive charges. The separately charged regions enable water to hydrate molecules that also have partially charged regions, which increases the solubility of the molecules.

The graph above shows the shell lengths of green sea turtle hatchlings from nests in either black volcanic sand or white calcareous sand beaches on a tropical island. The incubation temperatures of the nests were recorded. The error bars represent standard error. Which of the following statements best summarizes the data? A.Hatchling survival rates were higher in nests in black sand. B.Hatchling length decreased at higher temperatures in nests located in black sand or white sand. C.Eggs laid in nests in black sand were longer than eggs laid in nests in white sand. D.Hatchlings emerging from nests in black sand were statistically longer than those emerging from nests in white sand at all temperatures.

Correct Answer: B Option (B) is correct. When the egg‑incubation temperature was higher, the sea turtle hatchlings had shorter shells. The location and composition of the sand in which the nests were dug was irrelevant.

A population of beetles includes some individuals with light‑colored wings and other individuals with dark‑colored wings. A volcano erupts, and lava flow from the volcano covers much of the beetles' habitat. The frequency of beetles with dark wings increases significantly in the generations of beetles observed after the eruption. The wing‑color change is most consistent with A.disruptive selection B.directional selection C.stabilizing selection D.balancing selection

Correct Answer: B Option (B) is correct. When the organisms in a population initially display various forms of a trait including two extreme forms such as beetles with light‑colored and dark‑colored wings, but natural selection causes the average phenotype of the population to change in the direction of one extreme, the population is said to have undergone directional selection.

Chitin and cellulose are examples of A.energy-storage molecules B.information-storage molecules C.structural polysaccharides D.transmembrane channel proteins

Correct Answer: C Option (C) is correct. Chitin and cellulose are examples of structural polysaccharides. Chitin is found in the cell walls of many fungi and in the exoskeletons of arthropods and insects as well as in certain tissues of other animals. Cellulose is found in the cell walls of plant cells.

Which of the following structures is typically found in eukaryotic cells but not in prokaryotic cells? A.Chromosome B.Ribosome C.Mitochondrion D.Plasma membrane

Correct Answer: C Option (C) is correct. The mitochondrion is a membrane‑bound organelle that is required for aerobic cellular respiration in eukaryotes. Depending on the type of cell, the number of mitochondria per cell can vary from only one to a couple of thousand.

Which TWO of the following facts were critical to the demonstration by Alfred Hershey and Martha Chase that DNA rather than protein is the hereditary genetic material? A.Bacteriophages are more mobile than bacteria are. B.DNA contains phosphorus but protein does not. C.Bacteriophages insert only their genomes into bacteria. D.DNA can be labeled with two different isotopes of nitrogen.

Correct Answer: B, C Options (B) and (C) are correct. Alfred Hershey and Marsha Chase infected bacteria with bacteriophages, which are composed of a DNA genome surrounded by a protein coat. Because DNA contains phosphorus but protein does not, and protein contains sulfur but DNA D N Adoes not, Hershey and Chase were able to label the two components of the bacteriophage with two different isotopes and track which isotope(s) entered the bacteria to code for the production of new bacteriophage. The 32P P 32with which they labeled the bacteriophage DNA appeared inside the bacteria, and the 35S with which they labeled the bacteriophage protein remained in the supernatant outside the bacteria. This is because bacteriophages use their protein coats to adhere to receptors on the surface of bacteria and inject their DNA only into the bacteria as templates for the production of more bacteriophage by the bacteria.

Which TWO of the following are most likely to alter the allele frequencies of a small population of organisms over a short time? A.Crossing‑over during gamete formation B.Death of some members of the population from a disease C.An influx of individuals from an adjacent population D.Random mating among members of the population

Correct Answer: B, C Options (B) and (C) are correct. In a small population, the death of some members of the population can mean the loss of a large fraction of the population's copies of particular alleles. Similarly, the influx of individuals from an adjacent population can mean a relatively large increase in the fraction of particular alleles present or could result in the influx of new alleles of the genes, either of which will alter the allele frequencies.

A plant breeder cross‑pollinates a true‑breeding plant with red flowers with another true‑breeding plant of the same species that has white flowers. All of the F1 F 1plants have pink flowers. When any F1 F 1plant is cross‑pollinated with another F1 F 1plant, individual F2 F 2plants that are produced have red, white, or pink flowers. The data best support which of the following inheritance patterns for flower color in this species of plant? A.Codominance B.Mitochondrial C.Incomplete dominance D.Sex-linked

Correct Answer: C Option (C) is correct. An incomplete dominance inheritance pattern is suggested when both (1) the F1 F 1progeny of the cross of two true-breeding parents of distinct phenotypes are of an intermediate phenotype and (2) intercrossing the F1 F 1progeny results in F2 F 2organisms that collectively have all three phenotypes, those of the parental organisms or that of the F1 F 1. Typically, about 25% of the F2 F 2organisms have one of the parental phenotypes, 25% have the other parental phenotype, and 50% have the intermediate phenotype of the F1 F 1organisms.

A solution for an experiment requires 29.5 g gramsof NaCl. The following items are available for use in the preparation of the solution: a stock bottle of NaCl, a 1 L Erlenmeyer flask‚ a weigh boat‚ a digital balance, and a scoop. Which of the following is the most accurate way to prepare the solution, using only the items available? A.Tare the balance →→ place the weigh boat on the balance →→ use the scoop to add NaCl to the weigh boat B.Place the weigh boat on the balance →→ use the scoop to add NaCl to the weigh boat →→ tare the balance →→ place the flask on the balance →→ add the NaCl to the flask C.Place the weigh boat on the balance →→ tare the balance →→ use the scoop to add NaCl to the weigh boat D.Place the flask on the balance →→ tare the balance →→ use the scoop to add NaCl to the flask

Correct Answer: C Option (C) is correct. Because a 1 L flask has a narrow neck and is much heavier than the required amount of NaCl, the NaCl is weighed on a weigh boat rather than in the available flask. The weigh boat is placed on the balance, the balance is tared to subtract the weight of the weigh boat, and NaCl is transferred with the scoop from the stock bottle of NaCl to the weigh boat until the reading on the scale indicates that the correct mass has been added.

X‑linked severe combined immunodeficiency (SCID) is an inherited recessive disorder caused by a mutation to a gene located on the X chromosome. X‑linked SCID is likely to occur most often in A.females, because females have two X chromosomes and therefore have twice the chance of inheriting the mutant form of the gene B.females, because females have two X chromosomes and are therefore likely to have more severe disease than males C.males, because males have one X and one Y chromosome and therefore only require one mutated copy of the gene to exhibit the disease D.males, because males have two Y chromosomes and are therefore deficient for the gene

Correct Answer: C Option (C) is correct. Because males have only one X chromosome, males will have the phenotype determined by the one allele of any X‑linked gene they possess. Because females have two X chromosomes, females who are heterozygous for an X‑linked gene with a recessive form of inheritance will have the phenotype dictated by the dominant allele. Thus X‑linked SCID occurs much more frequently in males than in females.

In which of the following ways can birds most directly contribute to ecological succession in a particular area? A.Most birds can readily fly long distances from the area. B.Birds build nests from twigs they have found in the area. C.Birds drop seeds in their feces from plants they have eaten in surrounding areas. D.Genetic mutations enable evolution of the bird populations in the area.

Correct Answer: C Option (C) is correct. Birds eat the fruits and seeds of plants and then drop them in their feces. The seeds can germinate where they are dropped and thus introduce new plants to an area in a successional phase. If conditions are appropriate for the plants, they may spread throughout the area and contribute to ecological succession by altering the environment for other organisms.

Which of the following is the best example of habitat fragmentation? A.Marking a hiking trail through a forest B.Erecting a fire lookout tower where a forest meets a meadow C.Building a highway though a forest D.Scattering seeds of native wildflowers in a meadow

Correct Answer: C Option (C) is correct. Building a highway through a forest divides a continuous habitat into smaller isolated habitats. The result is that populations of animals and plants in the region may become fragmented and isolated as well. For many of the animal species, gene flow will be reduced, and some animals will be separated from their usual sources of food, water, and shelter. Similarly, there may be less cross-pollination between plants in the area. The presence of the highway also makes the animals more susceptible to injury and death both during and after highway construction.

Which of the following best describes the process of feedback inhibition of a metabolic pathway? A.Two enzymes that catalyze reactions in two different metabolic pathways compete for binding of the same substrate. B.An intermediate in the pathway is a substrate for an enzyme that catalyzes a later reaction in the pathway. C.An end product of the pathway reduces the activity of an enzyme that catalyzes an earlier reaction in the pathway. D.An unstable substrate that is required for a reaction in the pathway is the end product of a different metabolic pathway.

Correct Answer: C Option (C) is correct. Feedback inhibition of a metabolic pathway typically occurs when an end product of the pathway starts to build up and reduces the activity of an enzyme that functions early in the pathway. Such feedback inhibition often regulates the activity of allosteric enzymes that control branch points of a particular metabolic pathway. If the end product of one branch increases, the molecule will bind to the regulatory site of an allosteric enzyme that catalyzes an early step in that branch (feedback), thereby reducing the activity of that enzyme and concomitantly reducing the concentration of products of that branch of the pathway (inhibition).

Gregor Mendel proposed which of the following principles of genetics? A.Hybrid organisms have two different alleles on homologous chromosomes. B.Dominant genes occur more frequently in a population than recessive genes. C.Traits are passed in discrete units and produce predictable outcomes in offspring. D.An organism that is homozygous for many recessive traits is at a greater risk of disease.

Correct Answer: C Option (C) is correct. Gregor Mendel performed breeding experiments over several generations with pea plants that typically expressed only one of two possible phenotypes of a particular trait. By examining the phenotypes of the plants for many generations, he developed several basic principles of genetics, including one that states that traits are passed in discrete units (that we now know as alleles of genes) and produce predictable outcomes in offspring.

In the phylogenetic scheme shown‚ the clade marked by the arrow most likely represents which of the following taxa? A.Fungi B.Plantae C.Archaea D.Bacteria

Correct Answer: C Option (C) is correct. In the phylogenetic scheme shown, the three branches represent the three domains, with the arrow pointing to Archaea. Scientists currently think that the last universal common ancestor initially evolved into two lineages, Bacteria (the leftmost branch of the lineage shown) and another lineage (to the right) from which both the Archaea and Eukarya evolved. The proposed common origin of Archaea and Eukarya after the split from Bacteria was initially based on a sequence comparison of small subunit ribosomal RNA, an RNA that is particularly well conserved, from many organisms. One distinct difference between the Archaea and Eukarya is the presence of membrane‑bound organelles in Eukarya only. Thus the middle lineage in the phylogenetic tree shown must represent the Archaea.

Interphase is the part of the cell cycle that includes which of the following? A.Cytokinesis B.Synapsis of homologous chromosomes C.Replication of DNAD N A D.Condensation of chromosomes

Correct Answer: C Option (C) is correct. Interphase is the nonmitotic part of the cell cycle when a cell is growing and carrying out its normal metabolic processes. Replication of DNA D N Aoccurs during interphase, specifically during S phase of the cell cycle.

A particular anticancer drug binds to microtubules‚ preventing disassembly of microtubules into individual tubulin dimers. Based on this information, treatment with the drug most likely leads to the death of rapidly dividing cancer cells by interfering with which of the following processes? A.Oxidative respiration in mitochondria B.Actin filament attachments at the plasma membrane C.Mitotic spindle formation and function D.Messenger RNA R N Aprocessing and transport

Correct Answer: C Option (C) is correct. Mitotic spindles are composed of microtubules. Aligning the chromosomes at the equator of a mitotic cell requires both microtubule assembly and disassembly. Additionally, separating the chromatids to daughter cells involves microtubule disassembly. If the drug blocks microtubule disassembly, then it interferes with spindle formation and chromosome alignment and separation, blocking cell division. Such a drug typically manifests its greatest toxicity on rapidly dividing cells, including cancer cells.

Which of the following best explains why silent mutations in the coding region of a gene do not change the amino acid sequence of the protein encoded by the gene? A.Only the wild-type allele of the gene involved is transcribed; the allele with the mutation is not transcribed. B.Silent mutations are not translated because tRNAs do not bind to silent mutations in mRNAs and thus cause ribosomes to skip over the mutations. C.Most amino acids are encoded by several codons that differ by only one nucleotide, so some mutations do not change the encoded amino acid. D.RNA polymerases detect silent mutations in DNA and use their proofreading ability to transcribe RNAs of the correct sequence.

Correct Answer: C Option (C) is correct. Most, although not all, amino acids are encoded by more than one mRNA codon, with the difference between the codons frequently in the third base, the wobble position. A nucleotide change in this position is referred to as a silent mutation because it does not change the coded amino acid.

Which of the following is the best example of nonpoint source pollution in a lagoon? A.An algal bloom expanding in the lagoon B.An old car rusting in the shallow waters of the lagoon C.Fertilizers and pesticides carried in runoff from agricultural lands into the lagoon D.Raw sewage from a nearby community flowing through a pipe into the lagoon

Correct Answer: C Option (C) is correct. Nonpoint source pollution is typically caused by precipitation washing pollutants such as fertilizers and pesticides from the ground into bodies of water such as rivers, lakes, wetlands, lagoons, and coastal waters. The pollutants gradually accumulate in the water and are either directly or indirectly toxic to organisms.

Which of the following represents the largest carbon sink on Earth? A.Tropical forests B.Lakes C.Oceans D.Permafrost

Correct Answer: C Option (C) is correct. Oceans are the largest active carbon sink on Earth. The vast majority of the CO2 C O 2on Earth is stored in algae, vegetation, and coral in the oceans, but with the increasing acidity of the oceans, less CO2 C O 2can be absorbed by them than before the huge increase in industrial CO2 C O 2emissions.

Which of the following is the sexually mature, diploid structure of flowering plants? A.Embryo sac B.Pollen grain C.Sporophyte D.Gametophyte

Correct Answer: C Option (C) is correct. The life cycle of a flowering plant is characterized by alternation of generations, with a dominant sporophyte generation. The sporophyte is the diploid, sexually mature generation of flowering plants, and it produces haploid spores by meiosis. The spores divide by mitosis to produce the multicellular gametophyte generation that produces gametes.

Which of the following best explains why the amount of available energy is reduced with increasing trophic levels in an energy pyramid? A.Organisms are larger at higher trophic levels and store more energy than do organisms at lower trophic levels. B.Organisms at higher trophic levels require more energy per kg kilogramthan do organisms at lower trophic levels. C.Energy is lost as heat by organisms at each trophic level. D.There are fewer organisms at higher trophic levels than at lower trophic levels.

Correct Answer: C Option (C) is correct. Only chemical energy that is stored as biomass is passed on to the next trophic level; this is only about 10% of the energy in each trophic level. On average, about 90% of the energy in each trophic level is given off as heat during the cellular respiration of the organisms at that trophic level or in their feces that are then consumed by detritivores. The heat given off represents energy lost to the ecosystem.

Of the following, which is most likely defective in a person who is not getting enough oxygen delivered to his or her tissues? A.Platelets B.Plasma C.Hemoglobin D.White blood cells

Correct Answer: C Option (C) is correct. Oxygen in the blood is carried bound to the hemoglobin found in red blood cells. Of the options given, the defect in someone whose tissues are not sufficiently oxygenated is probably in hemoglobin.

Which of the following mutations in the genome of a fertilized ovum is likely to have the most deleterious effect on the early development of an embryo? A.A three-nucleotide insertion in a telomere B.A three-nucleotide deletion in the promoter of a gene C.A point mutation in a gene encoding a ribosomal RNA D.A point mutation in the middle of an intron of a gene

Correct Answer: C Option (C) is correct. Ribosomes are composed of specific ribosomal RNAs and proteins.A mutation of a gene encoding a ribosomal protein may alter the structure of the ribosomal protein and thus of all ribosomes in the ovum. This will, in turn, affect the synthesis of all proteins in the ovum. Such a deletion therefore is likely to have a more deleterious effect than a mutation of any other single gene in the ovum.

Which of the following best describes scientific evidence? A.It proves or disproves a belief. B.It can be accepted as fact. C.It can be used to support or reject a hypothesis. D.It allows a scientific theory to be viewed as a law.

Correct Answer: C Option (C) is correct. Scientific evidence is data gathered from experiments or observations to address a specific hypothesis. The data may help to support or reject the hypothesis.

Hillside agricultural practices such as terracing crops and planting crops in a contour pattern are primarily designed to limit the occurrence of which of the following? A.Flooding B.Infestation by insect pests C.Soil erosion D.Growth of weed species

Correct Answer: C Option (C) is correct. Terracing of crops and planting crops in a contour pattern are typically done when crops are grown on hills and are designed to slow water runoff and limit soil erosion from the hillsides after heavy rains.

At year 0 in the graph shown above, a small herd of deer was introduced to an area protected from human activity. The population was monitored for 60 years. Based on the curve, which of the following is closest to the carrying capacity of the area for deer? A.300 B.400 C.550 D.750

Correct Answer: C Option (C) is correct. The carrying capacity is the maximum population size that can be supported by resources available in the area. The deer population initially reached about 750 members, but it dropped down to as few as 400 members until it rebounded and stabilized at about 550 members. Although it continued to periodically increase above 550 members, it always dropped back down to about this size, presumably because of limiting resources.

Which of the following best supports the endosymbiotic theory? A.The similar embryonic development of organisms that belong to different taxonomic classes B.The codependence of certain organisms and other organisms that live within them C.The structure and function of chloroplasts D.The targeting of proteins to particular organelles

Correct Answer: C Option (C) is correct. The endosymbiotic theory proposes that an ancestral eukaryote ingested an aerobic nonphotosynthetic prokaryote and thus acquired mitochondria and that a descendant of the mitochondrion‑bearing eukaryote ingested a photosynthetic prokaryote and thus acquired chloroplasts. The structure and function of chloroplasts supports the endosymbiotic theory. Chloroplasts have a double membrane and ribosomes, and they reproduce independently of host cell division. They also contain circular DNA D N Athat encodes some of their own tRNAs, t R N A'sribosomal components, and proteins required for chloroplast function.

Which of the following human activities has been responsible for adding the greatest amount of carbon dioxide to the atmosphere over the last 300 years? A.Increased human respiratory output because of the increase in human population size B.Increased clear‑cutting of forests for agriculture C.Increased combustion of fossil fuels for energy needs D.Increased use of propellants that disrupt ozone levels in the atmosphere

Correct Answer: C Option (C) is correct. The increased combustion of fossil fuels, particularly with the start of the Industrial Revolution in approximately the mid‑1700s, seventeen hundredsis responsible for adding the greatest amount of carbon dioxide to the atmosphere over the last 300 years.

A researcher hypothesizes that lectures do not convey information as well as written passages in a textbook do. The researcher separates 48 college freshman volunteers into two groups of 24, with an equal number of males and females in each group. One group hears a passage repeated three times over a two‑minute period. The second group reads the same passage for two minutes. All test subjects are then asked to write down as much as they remember of the passage. Which of the following is the independent variable in the experiment? A.The number of males and females per group B.The length of the passage C.The method by which the passage is presented to the test subjects D.The accuracy and completeness of recall of the passageCorrect Answer: C Option (C) is correct. The independent variable in an experiment is the one thing that is varied between experimental groups. The effect of the independent variable is typically what is being analyzed. In the situation described, the independent variable is the method by which the passage is presented to the test subjects, either aurally or visually.

Correct Answer: C Option (C) is correct. The independent variable in an experiment is the one thing that is varied between experimental groups. The effect of the independent variable is typically what is being analyzed. In the situation described, the independent variable is the method by which the passage is presented to the test subjects, either aurally or visually.

In humans, which of the following combinations of a stimulus and a hormone released in response to the stimulus interact in a positive feedback loop? A.Rising blood glucose levels . . insulin B.Decreasing thyroid hormone levels . . thyroid-stimulating hormone C.Breastfeeding . . oxytocin D.Ingestion of salty food . . antidiuretic hormone

Correct Answer: C Option (C) is correct. The suckling of an infant triggers oxytocin secretion, which stimulates the contraction of the myoepithelial cells around the mammary glands, which in turn causes milk to squirt into the infant's mouth. This enables the continued suckling of the infant, which continues to trigger oxytocin secretion, forming a positive feedback loop.

If a messenger RNA contains the codon 5'5'-UCA- 3'3', the corresponding nucleotide sequence in the DNA template strand is most likely which of the following? A.5'5'-UGA- 3'3' B.5'5'-AGU- 3'3' C.5'5'-TGA- 3'3' D.5'5'-AGT- 3'

Correct Answer: C Option (C) is correct. The template strand for transcription of a DNA sequence to mRNA runs antiparallel to the mRNA and utilizes thymine (T) in the place of the uracil (U) found in RNA. If the sequence 5'−UCA− 3' is found in an mRNA, then the template DNA sequence must be 5'−TGA− 3'.

In a particular species of rodent, the predominant fur color of one population is tan, whereas the predominant fur color of another physically isolated population is dark brown. Loss of a physical barrier between the populations enables interbreeding between the populations. Which of the following is a likely long‑term outcome with respect to rodent fur coloration? A.Both color phenotypes will continue to exist in equal numbers. B.The color of the more prevalent population at the time of barrier loss will predominate. C.The color that provides better camouflage in the new environment will predominate. D.The color of the majority of breeding females will predominate.

Correct Answer: C Option (C) is correct. The two different rodent populations are most likely of different fur colors as a result of natural selection. Selective pressure to avoid predation would lead to coloration that provides better camouflage. Camouflage is likely to remain a major factor in the determination of rodent coloration after loss of the physical barrier and subsequent interbreeding of individuals.

Trisomy can cause conditions such as Down syndrome because the individual's cells A.are missing a chromosome B.are missing a gene C.have an additional chromosome D.have an additional nucleus

Correct Answer: C Option (C) is correct. Trisomy refers to the presence of an extra copy of a chromosome or most of a chromosome. It most frequently occurs when homologous chromosomes or sister chromatids fail to separate during meiosis, so that a gamete is formed with two copies, rather than one, of a particular chromosome. Fertilization of the gamete with a gamete containing the normal number of chromosomes results in a zygote with three, rather than two, copies of the particular chromosome and thus an extra set of the genes on that chromosome.

Under which of the following conditions is speciation most likely to occur? A.Environmental conditions in an area remain constant for an extended period of time. B.Offspring produced by a mating between two animals are phenotypically different from the parents. C.A physical barrier such as a river fragments a formerly widespread species into two populations. D.Available resources for a population are unlimited, and the individuals have high fitness.

Correct Answer: C Option (C) is correct. When a physical barrier completely separates members of a single population into two separate populations, speciation may occur. If the environmental influences and selective pressures differ for the two populations, the organisms may sufficiently change over time such that when organisms from the two populations are brought together, they can no longer produce fertile offspring and are considered to be separate species.

Which of the following best explains how insulin helps to regulate blood glucose levels in a healthy individual? A.It causes the hydrolysis of glycogen into glucose when body cells require a source of energy. B.It causes the kidneys to filter excess glucose from the blood into the urine. C.It causes the uptake and storage of excess glucose by cells of the body. D.It causes the cells of the pancreas to synthesize glycogen when blood glucose levels are too high.

Correct Answer: C Option (C) is correct. When the concentration of glucose in the blood increases, the beta cells of the pancreas secrete insulin. Insulin causes the uptake of glucose from the blood by cells of the body, especially by liver, skeletal muscle, and adipose cells.

Predictive testing is used to determine whether an individual has a predisposition for the onset of a particular disorder as a result of a genetic mutation. Predictive testing is generally performed when familial linkage for a disorder is suspected. Which TWO of the following are most likely to be ethical issues related to predictive testing? A.Only certain school-age children, based on information obtained through predictive testing, being required to get an annual medical checkup before participating in extracurricular sports B.Adoptive parents electing to have predictive tests of their child based on information in the adoption records C.Privacy violations resulting from information obtained through predictive testing D.Differential health insurance costs based on predictive testing results

Correct Answer: C, D Options (C) and (D) are correct. Two ethical issues most likely to arise from predictive testing involve privacy violations from information obtained through the testing and then inappropriately shared with others and inappropriate treatment of certain individuals by insurance companies based on the potential health problems uncovered by the testing. In an attempt to prevent discrimination against individuals based on the results of genetic testing, the Genetic Information Nondiscrimination Act was passed in 2008.

Which of the following terms collectively describes the interacting turtles‚ fish‚ insects‚ frogs, birds, and water plants that live in and around the same pond? A.An ecosystem B.A biome C.A population D.A community

Correct Answer: D Option (D) is correct. A community refers to the populations of all species in a particular area that can potentially interact with each other.

One estimate of the average mutation rate in the human nuclear genome is 0.5 mutations out of every 10^10 base pairs per replication. How is the mutation rate expressed in scientific notation? A.0.5 × 10^10 B.5 × 10^-9 C.5 × 10^-10 D.5 × 10^-11

Correct Answer: D Option (D) is correct. A mutation rate in the human nuclear genome of 0.5 mutations out of every 1010 ten to the tenbase pairs per replication is the same as a mutation rate of 5 ×× 10-11.

Which of the following is most likely to cause a species to become extinct in an environment undergoing change? A.A high mutation rate B.A low mutation rate C.High genetic diversity D.Low genetic diversity

Correct Answer: D Option (D) is correct. A species is most likely to survive in an environment undergoing change if there is genetic diversity among members of the species. Those individuals with a genetic background that best enables them to survive and reproduce in the changing environment will do so. If the species instead has low genetic diversity, there is a greater chance that a change in the environment will impact the species more severely, and the species may become extinct.

The incidence of managed fires and wildfires is routinely monitored and recorded in Yellowstone National Park. Data recorded between 2001 and 2010 are shown above. Based on the fire data, which of the following conclusions is most likely correct? A.The number of fires has a direct correlation with the number of acres burned. B.Firefighters always suppress more fires than they manage. C.Fires in Yellowstone National Park are mainly the result of human action. D.Fires in Yellowstone National Park are a normal occurrence.

Correct Answer: D Option (D) is correct. Based on the number of wildfires recorded for each of the years in the table, it appears that fires in Yellowstone National Park are a routine occurrence.

The flowers of many agave plants are short and tubular, are cream to dull yellow in color, have a pungent musky scent, and accumulate large amounts of nectar at night. Which of the following is the most likely pollination strategy of agave plants with such flowers? A.Pollination by wind B.Pollination by bees C.Pollination by hummingbirds D.Pollination by bats

Correct Answer: D Option (D) is correct. Because the flowers of agave plants are light rather than brightly colored, have a pungent musky scent, and accumulate nectar at night, they are primarily pollinated by nocturnal organisms that are attracted to such a scent and have physical adaptations that enable them to get to the nectar at the base of a tubular flower. Nectar is the main food source for many species of bats that are attracted to a musky scent, have relatively large eyes with which to detect light‑colored flowers, and have long tongues with which to reach the nectar at the base of the flower. As the bats feed, pollen sticks to their fur and is thus transferred between flowers.

Which of the following is an example of intraspecific competition? A.A cheetah and a lion in competition for a gazelle B.Sharks and killer whales in competition for seals C.Grasshoppers and caterpillars in competition for certain plants D.Two male red deer in competition for a mate

Correct Answer: D Option (D) is correct. Competition between members of a single species for resources is referred to as intraspecific competition. In this situation, the female mate is the resource for which two male deer compete.

Which of the following are most useful for determining which two species within a group of species share the most recent common ancestor? A.Primitive characters B.Behavioral characters C.Analogous characters D.Derived characters

Correct Answer: D Option (D) is correct. Derived characters are those characters that evolved in a particular lineage leading to a clade. The derived characters distinguish the species in that clade from the species in any other clade and indicate that they shared the most recent common ancestor.

Genes are located on which of the following? A.Centromere B.Codon C.Centriole D.Chromosome

Correct Answer: D Option (D) is correct. Genes are sequences of nucleotides on a chromosome. A gene contains sequence information for the synthesis of one or more related polypeptides or functional RNA R N Amolecules plus the regulatory sequences required for transcription of the gene.

Which of the following has most contributed to the recovery of gray wolf populations in the United States? A.Increased protection of domestic livestock from wolf attacks B.Release of minimally aggressive wolf-dog hybrids into wolf habitats C.Removal of species such as grizzly bears that compete for food resources in wolf habitats D.Reintroduction of wolves to protected areas such as Yellowstone National Park

Correct Answer: D Option (D) is correct. Gray wolves have been reintroduced to select areas in the western United States where gray wolves were common before they were all extirpated. Protected areas, such as areas in Yellowstone National Park, contain large tracts of wilderness and sufficient resources to allow the introduced wolves to thrive and reestablish populations.

A student is splattered in the face and eyes with some preservative during a frog dissection. Which of the following is the most appropriate immediate action for the student to take? A.Patting the eyes and interior of the eyelids dry with sterile gauze pads B.Patting the face and eyelids with paper towels to absorb the preservative C.Rinsing the face several times over a one-hour period with a sterile saline solution D.Rinsing the eyes and interior of the eyelids with water at an eyewash station for fifteen minutes

Correct Answer: D Option (D) is correct. If a student is splattered in the face with preservative, particular care must be taken to ensure that the student's eyes are washed free of any preservative. This is most successfully accomplished by having the student rinse the eyes and interior of the eyelids with water at an eyewash station, typically for fifteen minutes.

An autocrine hormone secreted by particular cells of an animal acts on which of the following? A.Different types of cells in the same tissue as the cell by which it was secreted B.Cells in body tissues that are distant from the hormone-secreting cell C.Sex hormone-secreting cells of another animal with which the animal mates D.The same cells by which the hormone was secreted

Correct Answer: D Option (D) is correct. In autocrine signaling, the hormones or factors that are secreted by a cell bind to receptors on the same cell and trigger a response by that cell.

Which of the following best explains why the pyramid of biomass for many lake ecosystems is inverted? A.The primary producers are larger in size than the primary consumers. B.The standing crop of primary producers is many times larger than the standing crop of primary consumers. C.There are no secondary consumers in the lakes, so the primary consumer populations become very large in size. D.The primary producers both reproduce and are eaten rapidly by primary consumers, so the primary producers do not accumulate in large numbers.

Correct Answer: D Option (D) is correct. In lakes, the primary producers are mostly phytoplankton, and the primary consumers are zooplankton. Even though the net energy production of the phytoplankton is far greater than the net energy production of the zooplankton, the standing biomass of the phytoplankton is significantly smaller than the standing biomass of the zooplankton. This is because the phytoplankton have a very short turnover time; they grow, reproduce, and are eaten rapidly by the zooplankton so that they do not accumulate any significant biomass.

Which of the following describes an experiment performed by Louis Pasteur in which he disproved the notion that living organisms arise by spontaneous generation? A.Several days after meat was placed in two separate containers, maggots were found on the meat in the open container, but in the container covered with mesh cloth, maggots were found only on the cloth. B.Bacteria grew from an animal's sputum sample that was spread on an agar plate, and injection of some of the bacteria into a second animal caused an infection similar to that in the first animal. C.Milk collected from a cow and placed directly in an open jug became sour after a couple of days, but milk that was heated at a high temperature for a short period of time took over a week to become sour in an open jug. D.Sterile meat broth in a flask with a straight neck quickly became cloudy when exposed to the air, but sterile meat broth in a flask with an S-shaped neck remained clear when exposed to air for the same length of time.

Correct Answer: D Option (D) is correct. Louis Pasteur performed a now‑classic experiment to test the hypothesis of spontaneous generation. He placed meat broth in each of two flasks with S‑shaped necks. He boiled the broth in both flasks to sterilize it and then broke off the S‑shaped portion of the neck from one flask. The broth in the flask that now had a straight neck became cloudy, while the broth in the flask with the S‑shaped neck remained clear because any airborne organisms were trapped in the neck of the flask.

Which of the following correctly matches a phylum with the most common mode of excretion by organisms in the phylum? A.Echinodermata . . flame cells B.Cnidaria . . nephrons C.Nematoda . . malpighian tubules D.Annelida . . metanephridia

Correct Answer: D Option (D) is correct. Members of the phylum Annelida (segmented worms such as earthworms) have metanephridia through which they excrete nitrogenous wastes. A pair of metanephridia in each segment of the worm remove the wastes from the surrounding coelomic fluid.

Which of the following situations is an example of primary ecological succession? A.A volcanic eruption destroys 300 acres of coniferous forest in the fall. The following spring, park rangers seed grasses and wildflowers. B.A forest fire burns out the undergrowth in a chaparral biome. After the next rainfall, new buds on the shrubs that survived the fire begin to open. C.A violent storm causes many trees in an area to fall over. Over time, plant species that differ from the fallen trees become the dominant species. D.A lava flow from a vent on the ocean floor creates a new island. Over time, seeds and spores blown onto the island germinate, take root, and grow.

Correct Answer: D Option (D) is correct. Primary ecological succession typically begins in an area that is free of soil or living organisms. This contrasts with secondary succession that typically occurs in an area that has been cleared of most organisms by a disturbance that leaves the soil intact. An island created from a lava flow is expected to initially have no soil or living organisms. Seeds and spores that blow onto the island will find cracks and crevices and perhaps loose lava where they can germinate, take root, and grow. Their growth will change the nature of the lava, allowing seeds and spores from ferns and other small plants to take root. Over time, as soil develops, portions of the island will be covered with low vegetation and eventually with a more complex array of vegetation.

Which of the following enzyme types is most likely responsible for the cleavage of the plasmid shown above? A.Ligase B.Helicase C.DNA D N Apolymerase D.Restriction endonuclease

Correct Answer: D Option (D) is correct. Restriction endonucleases are used to cleave DNA. D N AMany recognize and cleave at palindromic sequences and create nucleotide overhangs such as that shown in the figure of the cleaved plasmid.

Serum albumin, a blood protein produced by hepatocytes in the liver, has a molecular mass of approximately 66.5 kilodaltons (kDa). k D ABy which of the following processes is newly synthesized albumin typically transferred from the cytosol of a hepatocyte to the blood? A.Passive diffusion B.Cell lysis C.Phagocytosis D.Exocytosis

Correct Answer: D Option (D) is correct. Secreted proteins such as serum albumin are released from cells by exocytosis. A protein that is destined for secretion enters the rough endoplasmic reticulum (ER) E Ras it is translated, then is moved in a vesicle from the ER E Rto the Golgi apparatus, where it is transported in vesicles from one cisterna to the next, and finally a secretory vesicle that contains the protein fuses with the plasma membrane, releasing the protein from the cell.

The Hardy-Weinberg equilibrium equation is most useful for which of the following? A.Determining the conditions that must be set up in an experimental population to ensure that evolution does not occur B.Determining the frequency with which mutations occur in a population to test whether the mutations are harmful C.Quantifying the impact of sexual reproduction on a population D.Calculating the frequency of a harmful allele in a population

Correct Answer: D Option (D) is correct. The Hardy‑Weinberg principle states that genotype frequencies in a population remain constant as long as the population is large and not subject to any evolutionary processes that provide selective pressure. The Hardy‑Weinberg equilibrium equation can be applied to such a population to calculate the frequency of a particular allele of a gene that has only two alleles. The equation states that p2 + 2pq + q2 = 1, where p is the frequency of one allele of the gene and q is the frequency of the other allele of the gene. p2 p squaredis the percent of individuals that are homozygous for the first allele, q2 2 p qis the percent of individuals that are homozygous for the second allele, and 2pq q squaredis the percent of heterozygous individuals. If the percent of the population that expresses a particular allele (e.g., q2 q squared) is known, then the frequency of the allele in the population can be determined.

Which of the following best explains why the roots of garlic bulbs must be completely removed before the bulbs are imported into the United States? A.The garlic is more likely to sprout new roots, which increases the likelihood of healthy growth. B.The garlic is less likely to absorb taste-altering contaminants in the air. C.The garlic is more likely to have a long shelf life in grocery stores. D.The garlic is less likely to carry foreign root pathogens.

Correct Answer: D Option (D) is correct. The United States Department of Agriculture requires that all roots be removed from garlic bulbs entering the United States to prevent the entry of soilborne plant pathogens on the roots.

A difference in which of the following is most likely to be a prezygotic barrier to successful hybridization between two species? A.Mitochondrial DNA D N Asequence B.Amount of heterozygosity C.Length of gestation D.Acrosomal proteins

Correct Answer: D Option (D) is correct. The acrosome is a caplike structure on the head of a sperm. It contains enzymes used in the acrosomal reaction when the sperm digests its way through the zona pellucida that covers and protects the surface of an ovum, so that the plasma membrane of the sperm can fuse with the plasma membrane of the ovum. The acrosomal proteins and zona pellucida usually differ between species. If the ovum and sperm are of different species, an acrosomal reaction will not occur between the sperm and the ovum, preventing fertilization. Thus a difference in the acrosomal proteins of two species is a prezygotic isolating mechanism.

ACE inhibitors are a class of drugs that lower blood pressure by inhibiting angiotensin converting enzyme (ACE)‚ which converts angiotensin I1 into angiotensin II‚2 a substance that increases blood pressure. To investigate whether drug X is a competitive or noncompetitive inhibitor of ACE‚ a series of reactions were performed in the absence or presence of drug X, and the relative amount of angiotensin II2 produced in each reaction was determined. Each reaction contained the same amount of enzyme, was adjusted to the same volume and buffer conditions, and was carried out for the same length of time. The results are shown in the figure. Which of the following statements is best supported by the data? A.Drug X is a noncompetitive inhibitor, because the rate of reaction in the presence of the inhibitor is slower at all substrate concentrations. B.Drug X is a noncompetitive inhibitor, because the enzyme only functions at the highest concentration of substrate in the presence of the inhibitor. C.Drug X is a competitive inhibitor, because less product is formed in the presence of the inhibitor at all substrate concentrations. D.Drug X is a competitive inhibitor, because the same amount of product is formed in the presence and absence of the inhibitor at the highest substrate concentration.

Correct Answer: D Option (D) is correct. The amount of product, angiotensin II,2 formed in a period of time allows the calculation of the rate of angiotensin II2 formation that is plotted in the graph. In the absence of drug X, the rate of angiotensin II formation increases with an increasing amount of the substrate angiotensin I1 until the rate plateaus at 3 mMmillimolar angiotensin I,1 when the enzyme active sites are most likely saturated with substrate. In the presence of drug X, the rate of angiotensin II2 formation is decreased at low concentrations (1 mMmillimole and 2 mMmillimoles) of the substrate angiotensin I,1 but there is very little to no decrease in angiotensin II2 formation by drug X when angiotensin I is used at higher concentrations (3 mMmillimoles and (4 mMmillimoles). Drug X must compete for the enzyme's active site because, in the presence of a constant amount of drug X and an increasing concentration of angiotensin I,1 the rate of angiotensin II2 formation at the highest concentrations of angiotensin I1 is approximately the same in the absence or presence of drug X.

Some species of hermit crab frequently camouflage their shells with sea anemones. The anemones receive food scraps and transportation from the crabs. The relationship between the hermit crabs and the anemones is best described as A.parasitism B.competition C.commensalism D.mutualism

Correct Answer: D Option (D) is correct. The arrangement between the hermit crabs and the sea anemones is beneficial to both organisms. Thus the relationship between them is best described as mutualism.

The ecological footprint of a country is a measure of the A.biological capacity of ecosystems to regenerate natural resources B.consumption of carbon and water by the populations of an ecosystem C.human utilization of carbon relative to the rate of climate change D.amount of productive land and sea required to supply the needs of the entire population

Correct Answer: D Option (D) is correct. The ecological footprint of a country refers to the impact of the country on the environment and measures the amount of productive land and sea required to provide the resources for the entire population of the country. Calculation of the ecological footprint can be used to analyze the sustainability of the population of the country under current conditions.

The pedigree above illustrates inheritance of a rare trait in a family. The mode of inheritance of the trait is most likely which of the following? A.Autosomal recessive B.Autosomal dominant C.X-linked recessive D.X-linked dominant

Correct Answer: D Option (D) is correct. The mode of inheritance is most likely X‑linked dominant. A female who expresses the trait in any generation passes it on to approximately half her children, irrespective of the child's sex. A male who expresses the trait passes it on to all of his female children but not to any of his male children. This suggests that the trait is encoded by a gene on the X chromosome. Because the trait is said to be rare, those without the trait are unlikely to carry an allele for the trait. Since the trait appears in every generation and appears to be caused by a single allele from one parent, the allele causing expression of the trait is most likely dominant.

Of the following human activities, which is having the greatest negative impact on global biodiversity? A.Producing genetically modified organisms B.Using renewable energy sources C.Recycling consumer products D.Overharvesting native species

Correct Answer: D Option (D) is correct. The overharvesting of native species has led to the extinction or decimation of many species. Hunting of the passenger pigeon led to its extinction; overfishing of wild populations of fish has decimated the numbers of many fish species. The loss of any one animal species upsets the balance of the ecosystem of which the animal was a part and typically has a negative impact on the remaining species and on the biodiversity of the ecosystem.

The graph above shows the population sizes of two species of rodents‚ R and S‚ in a particular area over time. At the point indicated by the arrow‚ 500 individuals of species S were released into the area. Population sizes of the two species were monitored over the next 20 years. Based on the data‚ which of the following conclusions is most likely correct about the two rodent species in the area? A.Species S is being outcompeted by species R. B.Species S evolved quickly to adapt to the environment. C.The populations of species R and S have the same carrying capacity for the environment. D.Species R and S occupy different niches.

Correct Answer: D Option (D) is correct. The population size of rodent species S continues to increase for a period of time after its introduction into the area until it stabilizes at a fairly constant level, presumably the carrying capacity of the area for species S. Similarly, the population size of rodent species R remains fairly constant and is unchanged by the introduction of species S. Species R is probably also at carrying capacity. Based on the stability of the population sizes of the two rodent species and lack of apparent competition between the populations, the species most likely occupy different niches.

Familial hypercholesterolemia is an autosomal dominant disorder, and red-green color blindness is an X‑linked recessive trait. A woman with normal serum cholesterol levels who is color‑blind has several children with a man with familial hypercholesterolemia and normal vision. The man's father also had familial hypercholesterolemia, but his mother did not. Which of the following is closest to the probability that one of the children born to the man and woman will be a daughter with familial hypercholesterolemia and color blindness? A.1.0 B.0.50 C.0.25 D.0

Correct Answer: D Option (D) is correct. The probability that a child will be a daughter is 0.5. The probability that the daughter will have familial hypercholesterolemia is 0.5. The probability that the daughter will be color‑blind is 0, because the trait is recessive and requires two recessive alleles for expression in a female, but the father of the daughter has normal vision and thus no recessive allele to pass on. Thus, based on the family history, the probability that the man and woman will have a color‑blind daughter with familial hypercholesterolemia is 0.

Which of the following occurs during the processing of a eukaryotic pre-mRNA m R N Amolecule? A.The addition of a 5'5' cap to the pre-mRNA m R N Acauses transcription to terminate. B.Exons are spliced out of the pre-mRNA m R N A, and introns are joined to encode a polypeptide. C.Splicing of the pre-mRNA m R N Ainduces the binding of ribosomal subunits to the initiation codon. D.The pre-mRNA m R N Ais cleaved at its 3'3' end, and approximately 200 AMPs A M P'sare added to the cleaved end.

Correct Answer: D Option (D) is correct. The processing of a eukaryotic pre-mRNA m R N Ato an mRNA includes the cleavage of a portion of the pre-mRNA m R N A3'3' to the AAUAAA sequence that is located near the 3'3' end of the pre-mRNA m R N A, followed by polyadenylation, the addition by a particular RNA R N Apolymerase of about 200 AMP A M Presidues to the cleaved end.

Which of the following plant structures most likely contains sclerenchyma cells? A.The primary phloem of a sunflower plant B.The corolla of a hibiscus flower C.The apical meristem within the terminal bud of a birch tree branch D.The pericarp of a hazelnut

Correct Answer: D Option (D) is correct. The shell of nuts such as a hazelnut is the pericarp, the thickened wall of the angiosperm ovary. Sclerenchyma cells have thick secondary cell walls that are strengthened by lignin. One type of sclerenchyma cell, the sclereid, is responsible for the hard structure of the nut pericarp.

Which of the following occurs as a part of the noncyclic photophosphorylation pathway of photosynthesis? A.Protons (H+) H with a positive chargeflow through ATP synthase into the thylakoid space. B.NAD+ N A D with a positive chargeis reduced to NADH N A D Hby electron transfer from ferredoxin. C.Electrons are transferred from photosystem I 1to photosystem II.2 D.Water is split to two electrons, two protons, and an oxygen atom.

Correct Answer: D Option (D) is correct. The source of electrons for photosystem II 2(PS II), P S 2the first photosystem activated in the Z scheme of noncyclic photophosphorylation, is water. When pigments in the light‑harvesting complex of PS II P S 2absorb photons of light, resonance energy among the complex's pigment molecules enables the transfer of an electron from a chlorophyll a molecule to the primary electron acceptor of PS II P S 2and then to an electron transport chain. As a result of the electron loss, the oxidized chlorophyll a molecule becomes so electronegative that enzymes in the photosystem can strip electrons from water to replace the lost electron. The oxygen atoms from two water molecules combine to produce the O2 O 2that is released by plants during photosynthesis. The splitting of water and the associated oxygen‑generating reaction can be written as: 2 H2O →→ 4 H+ + 4 e− + O2.

A student uses stationary fans to investigate the effect of wind on water loss by plants. The student divides the plants into four treatment groups and places each group in front of a fan. The student sets the fan for each group to one of four different levels: low, medium, high, or OFF. After 24 hours, the student determines the amount of water that was lost by the plants. To create a bar graph of the results, the student should make the height of each bar represent which of the following? A.The fan speed for each treatment group B.The number of plants in each treatment group C.The duration of wind exposure for each treatment group D.The average water loss for each treatment group

Correct Answer: D Option (D) is correct. There are four separate treatment groups, so each group can be represented by a separate bar on the x‑axis of a graph where the y‑axis is marked in increments of the average volume of water lost over 24 hours. The student should determine the amount of water lost by each plant in each group then average the data for all plants in a single group and draw a bar to represent the average water lost by the plants in that group.

Which of the following is a typical characteristic of vertebrates that reproduce through external fertilization? A.They reach sexual maturity at a young age. B.They produce male and female gametes that are similar in size. C.They live in a terrestrial environment. D.They release a large number of gametes at once.

Correct Answer: D Option (D) is correct. Vertebrates that reproduce through external fertilization typically release an enormous number of gametes into an aqueous environment to meet the challenge of ensuring that a sufficient number of eggs are fertilized by sperm before the gametes randomly disperse.

Strawberry plants can reproduce asexually by sending out runners that root in the soil or sexually by cross‑pollination between different plants or between male and female flowers on the same plant. Under which of the following conditions is asexual reproduction likely to be more effective than pollination for production of additional strawberry plants? A.In springtime when pollinators are numerous and water is plentiful B.In springtime in an area where overcrowding is causing competition for water and nutrients C.In midsummer when the plant is affected by a pathogen for which it has no genetic defense D.In midsummer when space is adequate but water and pollinators are in low supply

Correct Answer: D Option (D) is correct. When a plant reproduces by sending out runners that root in the soil and grow into new plants, the new plants initially obtain nutrients that are distributed through the runners from the parent plant. The new plants are less dependent on water than would be seeds produced through sexual reproduction that require water for germination. Similarly, growth of new plants from runners removes the requirement for pollinators.


संबंधित स्टडी सेट्स

Ch 46 Acute Kidney Injury and Chronic Kidney Disease

View Set

Intermediate Accounting II- CH 19 & 21

View Set

Chapter 15: Management of Patients with Oncologic Disorders

View Set

Ch. 8 Group and Team Effectiveness

View Set

Simple, Compound, Complex, Compound-Complex Sentences

View Set

Macroeconomics: Chapter 1 Homework

View Set

The Civil War Study Island Answers

View Set